summaryrefslogtreecommitdiff
path: root/Master/texmf-dist/doc/latex/jnuexam/exam-a.tex
blob: 98ff78f9e60155e2b7d87625a5d3420eab439c25 (plain)
1
2
3
4
5
6
7
8
9
10
11
12
13
14
15
16
17
18
19
20
21
22
23
24
25
26
27
28
29
30
31
32
33
34
35
36
37
38
39
40
41
42
43
44
45
46
47
48
49
50
51
52
53
54
55
56
57
58
59
60
61
62
63
64
65
66
67
68
69
70
71
72
73
74
75
76
77
78
79
80
81
82
83
84
85
86
87
88
89
90
91
92
93
94
95
96
97
98
99
100
101
102
103
104
105
106
107
108
109
110
111
112
113
114
115
116
117
118
119
120
121
122
123
124
125
126
127
128
129
130
131
132
133
134
135
136
137
138
139
140
141
142
143
144
145
146
147
148
149
150
151
152
153
154
155
156
157
158
159
160
161
162
163
164
165
166
167
168
169
170
171
172
173
174
175
176
177
178
179
180
181
182
183
184
185
186
187
188
189
190
191
192
193
194
195
196
197
198
199
200
201
202
203
204
205
206
207
208
209
210
211
212
213
214
215
216
217
218
219
220
221
222
223
224
225
226
227
228
229
230
231
232
233
234
235
236
237
238
239
240
241
242
243
244
245
246
247
248
249
250
251
252
253
254
255
256
257
258
259
260
261
262
263
264
265
266
267
268
269
270
271
272
273
274
275
276
277
278
279
280
281
282
283
284
285
286
287
288
289
290
291
292
293
294
295
296
297
298
299
300
301
302
303
304
305
306
307
308
309
310
311
312
313
314
315
316
317
318
319
320
321
322
323
324
325
326
327
328
329
330
331
332
333
334
335
336
337
338
339
340
341
342
343
344
345
346
347
348
349
350
351
352
353
354
355
356
357
358
359
360
361
362
363
364
365
366
367
368
369
370
371
372
373
374
375
376
377
378
379
380
381
382
383
384
385
386
387
388
389
390
391
392
393
394
395
396
397
398
399
400
401
402
403
404
405
406
407
408
409
410
411
412
413
414
415
416
417
418
419
420
421
422
423
424
425
426
427
428
429
430
431
432
433
434
435
436
437
438
439
440
441
% -*- coding: utf-8 -*-
% !TEX program = xelatex
\documentclass{jnuexam}

%\answerfalse %不显示答案

\setlength\arraycolsep{4pt}
\newcommand{\cov}{\operatorname{cov}}

\begin{document}

\renewcommand{\niandu}{2017--2018}
\renewcommand{\xueqi}{2}
\renewcommand{\kecheng}{大学数学}
\renewcommand{\zhuanye}{理工~4~学分} % 可以为空白
\renewcommand{\jiaoshi}{张三,李四,王五} % 教师姓名
\renewcommand{\shijian}{2018~年~06~月~28~日}
\renewcommand{\bixiu}{1} % 1 为必修,0 为选修
\renewcommand{\bijuan}{1} % 1 为闭卷,0 为开卷
\renewcommand{\shijuan}{A} % A 或 B 或 C 卷
\renewcommand{\neizhao}{1} % 1 打勾,0 不勾
\renewcommand{\waizhao}{0} % 1 打勾,0 不勾

\makehead % 生成试卷表头

\makepart{填空题}{共~8~小题,每小题~2~分,共~16~分}

\newpageb % B卷分页点

\begin{problem}
已知二阶行列式 $\text{$\left|\begin{array}{cc}
  1 & 2\\
  - 3 & x
\end{array}\right|$=0}$,则 $x=$ \fillout{$-6$}。
\end{problem}

\vfill

\begin{problem}
五阶行列式的一共有 \fillout{$120$} 项。
\end{problem}

\vfill

\begin{problem}
向量组 $\alpha_1=(1,1,0), \alpha_2=(0,1,1), \alpha_3=(1,0,1)$,
则将向量 $\beta=(4, 5, 3)$ 表示为 $\alpha_1, \alpha_2, \alpha_3$
的线性组合为 $\beta=$ \fillout{$3\alpha_1+2\alpha_2+\alpha_3$}。
\end{problem}

\vfill

\begin{problem}
已知$P(A)=0.3$, $P(B|A)=0.4$, $P(B|\bar{A})=0.5$, 则$P(B)=$ \fillout{$0.47$}。
\end{problem}

\vfill

\begin{problem}
已知连续型$\xi$的密度函数为$\varphi(x)=\left\{
\begin{array}{ll}
  k \cos x, & - \frac{\pi}{2} < x < \frac{\pi}{2}\\
  0, & \text{其它}
\end{array}\right.$,
则$k=$ \fillout{$\frac{1}{2}$}。
\end{problem}

\vfill

\begin{problem}
已知随机变量$\xi$的期望和方差各为$E\xi=3, D\xi=2$, 则$E\xi^2=$ \fillout{$11$}。
\end{problem}

\vfill

\begin{problem}
电子管寿命$\xi$满足平均寿命为$1000$小时的指数分布,则它的寿命小于$2000$小时概率为 \fillout{$1-e^{-2}$}。
\end{problem}

\vfill

\begin{problem}
已知$\xi$和$\eta$相互独立且$\xi\sim N(1,4), \eta\sim N(2,5)$,则$\xi-2\eta\sim$ \fillout{$N(-3,24)$}。
\end{problem}

\vfill

\newpagea % A卷分页点

\makepart{单选题}{共~8~小题,每小题~2~分,共~16~分}

\newpageb % B卷分页点

\begin{problem}
下列各排列哪个是偶排列 \pickout{D}
\quaritem{$3712456$}
\quaritem{$36715284$}
\quaritem{$654321$}
\quaritem{$41253$}
\end{problem}

\vfill

\begin{problem}
若三阶行列式 $\left|\begin{array}{ccc}
  a_1 & a_2 & a_3\\
  2 b_1 - a_1 & 2 b_2 - a_2 & 2 b_3 - a_3\\
  c_1 & c_2 & c_3
\end{array}\right| = 2$,则 $\left|\begin{array}{ccc}
  a_1 & a_2 & a_3\\
  b_1 & b_2 & b_3\\
  c_1 & c_2 & c_3
\end{array}\right|=$ \pickout{A}
\quaritem{$1$}
\quaritem{$-1$}
\quaritem{$2$}
\quaritem{$-2$}
\end{problem}

\vfill

\begin{problem}
已知矩阵 $A = \left(\begin{array}{ccc}
  1 & 1 & 0\\
  1 & x & 0\\
  0 & 0 & 1
\end{array}\right)$ 其中两个特征值为 $\lambda_1 = 1$ 和 $\lambda_2
= 2$,则 $x=$ \pickout{B}
\quaritem{$2$}
\quaritem{$1$}
\quaritem{$0$}
\quaritem{$-1$}
\end{problem}

\vfill

\begin{problem}
二次型 $f = 4 x_1^2 - 2 x_1 x_2 + 6 x_2^2$ 对应的矩阵等于 \pickout{C}
\quaritem{$\left(\begin{array}{cc}
  4 & - 2\\
  - 2 & 6
\end{array}\right)$}
\quaritem{$\left(\begin{array}{cc}
  2 & - 2\\
  - 2 & 3
\end{array}\right)$}
\quaritem{$\left(\begin{array}{cc}
  4 & - 1\\
  - 1 & 6
\end{array}\right)$}
\quaritem{$\left(\begin{array}{cc}
  2 & - 1\\
  - 1 & 3
\end{array}\right)$}
\end{problem}

\vfill

\begin{problem}
对任何一个本校男学生,以$A$表示他是大一学生,$B$表示他是大二学生,则事件$A$和$B$是\pickout{B}
\halfitem{对立事件}
\halfitem{互斥事件}
\halfitem{既是对立事件又是互斥事件}
\halfitem{不是对立事件也不是互斥事件}
\end{problem}

\vfill

\begin{problem}
下列说法\CJKunderline{不正确}的是\pickout{B}
\fullitem{大数定律说明了大量相互独立且同分布的随机变量的均值的稳定性}
\fullitem{大数定律说明大量相互独立且同分布的随机变量的均值近似于正态分布}
\fullitem{中心极限定理说明了大量相互独立且同分布的随机变量的和的稳定性}
\fullitem{中心极限定理说明大量相互独立且同分布的随机变量的和近似于正态分布}
\end{problem}

\vfill

\begin{problem}
在数理统计中,对总体$X$和样本$(X_1,\cdots,X_n)$的说法哪个是\CJKunderline{不正确}的\pickout{D}
\halfitem{总体是随机变量}
\halfitem{样本是$n$元随机变量}
\halfitem{$X_1, \cdots, X_n$相互独立}
\halfitem{$X_1 = X_2 =\cdots = X_n$}
\end{problem}

\vfill

\begin{problem}
样本平均数$\bar{X}$\CJKunderline{未必是}总体期望值$\mu$的\pickout{A}
\quaritem{最大似然估计}
\quaritem{有效估计}
\quaritem{一致估计}
\quaritem{无偏估计}
\end{problem}

\vfill

\newpagea % A卷分页点

\makepart{计算题}{共~6~小题,每小题~8~分,共~48~分}

\newpageb % B卷分页点

\begin{problem}
计算四阶行列式 $A = \left|\begin{array}{cccc}
  0 & 1 & 2 & 3\\
  1 & 2 & 3 & 0\\
  2 & 3 & 0 & 1\\
  3 & 0 & 1 & 2
\end{array}\right|$ 的值。
\end{problem}

\bigskip

\begin{solution}
$A = \left|\begin{array}{cccc}
    0 & 1 & 2 & 3\\
    1 & 2 & 3 & 0\\
    2 & 3 & 0 & 1\\
    3 & 0 & 1 & 2
  \end{array}\right| = \left|\begin{array}{cccc}
    0 & 1 & 2 & 3\\
    1 & 2 & 3 & 0\\
    0 & - 1 & - 6 & 1\\
    0 & - 6 & - 8 & 2
  \end{array}\right| = 1 \cdot (- 1)^{2 + 1} \left|\begin{array}{ccc}
    1 & 2 & 3\\
    - 1 & - 6 & 1\\
    - 6 & - 8 & 2
  \end{array}\right|$ \dotfill 4分\par
\qquad\qquad $= -\left|\begin{array}{ccc}
    1 & 2 & 3\\
    0 & - 4 & 4\\
    0 & 4 & 20
  \end{array}\right| = - \left|\begin{array}{cc}
    - 4 & 4\\
    4 & 20
  \end{array}\right| = -(-4\cdot20-4\cdot4) = 96$ \dotfill 8分
\end{solution}

\vfill

\begin{problem}
用配方法将二次型 $f = x_1^2 + 2 x_1 x_2 - 6 x_1 x_3 + 2 x_2^2 - 12
x_2 x_3 + 9 x^2_3$ 化为标准形 $f = d_1 y^2_1 + d_2 y^2_2 + d_3 y^2_3$ 。
\end{problem}

\bigskip

\begin{solution}
$f = x_1^2 + 2 x_1 x_2 - 6 x_1 x_3 + 2 x_2^2 - 12 x_2 x_3 + 9 x^2_3$ \par
\qquad\qquad$= x_1^2 + 2 x_1 (x_2 - 3 x_3) + (x_2 - 3 x_3)^2 + x_2^2 - 6 x_2 x_3 $ \par
\qquad\qquad$= (x_1 + x_2 - 3 x_3)^2 + x_2^2 - 6 x_2 x_3$ \dotfill 3分 \par
\qquad\qquad$= (x_1 + x_2 - 3 x_3)^2 + x_2^2 - 2 x_2 \cdot 3 x_3 + (3 x_3)^2 - 9x_3^2$ \par
\qquad\qquad$= (x_1 + x_2 - 3 x_3)^2 + (x_2 - 3 x_3)^2 - 9 x_3^2$ \dotfill 6分\par
令$y_1 = x_1 + x_2 - 3 x_3, y_2 = x_2 - 3 x_3, y_3 = x_3$, \newline
则$f = y_1^2 + y_2^2 - 9y_3^2$为标准形。\dotfill 8分
\end{solution}

\vfill

\newpage

\begin{problem}
设二元随机变量$(\xi, \eta)$的联合分布表为
\begin{tabular}{|l|l|l|l|}
  \hline
  $\xi \backslash \eta$ & -1 & 0 & 1\\
  \hline
  0 & 0 & 1/3 & 0\\
  \hline
  1 & 1/3 & 0 & 1/3\\
  \hline
\end{tabular}。\par
(1) 求关于$\xi$和$\eta$的边缘分布。\par
(2) 判断$\xi$和$\eta$的独立性。\par
(3) 判断$\xi$和$\eta$的相关性。
\end{problem}

\bigskip

\begin{solution}
(1) 边缘分布为 \begin{tabular}{|l|l|l|}
  \hline
  $\xi$ & 0 & 1\\
  \hline
  $P$ & 1/3 & 2/3\\
  \hline
\end{tabular}, \ \begin{tabular}{|l|l|l|l|}
  \hline
  $\eta$ & -1 & 0 & 1\\
  \hline
  $P$ & 1/3 & 1/3 & 1/3\\
  \hline
\end{tabular}. \dotfill 2分 \par
(2) 由$P(\xi = 0, \eta = 0) = \frac{1}{3} \neq \frac{1}{9} = P(\xi = 0) P(\eta = 0)$, 
知$\xi$和$\eta$不独立. \dotfill 4分 \par
(3) 由联合分布表求得$\xi \eta$的分布为 \begin{tabular}{|l|l|l|l|}
  \hline
  $\xi \eta$ & -1 & 0 & 1\\
  \hline
  $P$ & 1/3 & 1/3 & 1/3\\
  \hline
\end{tabular}.\dotfill 6分\par
因此有 $\cov(\xi, \eta) = E(\xi\eta) - E\xi E\eta = 0 -\frac{2}{3} \cdot 0 = 0$,
因此$\xi$和$\eta$不相关. \dotfill 8分
\end{solution}

\vfill

\begin{problem}
设随机变量$\xi \sim N (1, 4)$,求$P (- 1 < \xi < 5)$。
\end{problem}

\bigskip

\begin{solution}
$P(-1<\xi<5) = \Phi_0\left(\frac{5-1}{2}\right) - \Phi_0\left(\frac{-1-1}{2}\right)$ \dotfill 2分 \par
\qquad $= \Phi_0 (2) - \Phi_0 (- 1)$ \dotfill 4分 \par
\qquad $= \Phi_0 (2) + \Phi_0 (1) - 1$ \dotfill 6分 \par
\qquad $= 0.9773 + 0.8413 - 1 = 0.8186$ \dotfill 8分
\end{solution}

\vfill

\newpage % A,B卷共同分页点

\begin{problem}
设每发炮弹命中飞机的概率是0.2且相互独立,现在发射100发炮弹。\par
(1) 用切贝谢夫不等式估计命中数目$\xi$在10发到30发之间的概率。\par
(2) 用中心极限定理估计命中数目$\xi$在10发到30发之间的概率。
\end{problem}

\bigskip

\begin{solution}
$E\xi = n p = 100 \cdot 0.2 = 20, D\xi = n p q = 100 \cdot 0.2 \cdot 0.8 = 16$. \dotfill 2分 \par
(1) $P (10 < \xi < 30) = P (| \xi - E \xi | < 10) \geq 1 - \frac{D\xi}{10^2}
     = 1 - \frac{16}{100} = 0.84$. \dotfill 4分 \par
(2) $P (10 < \xi < 30) \approx \Phi_0 \left( \frac{30 - 20}{\sqrt{16}}\right)
     - \Phi_0 \left( \frac{10 - 20}{\sqrt{16}} \right)$ \dotfill 6分\par
\qquad $= 2 \Phi_0 (2.5) - 1 = 2 \cdot 0.9938 - 1 =0.9876$ \dotfill 8分
\end{solution}

\vfill

\begin{problem}
从正态总体$N(\mu,\sigma^2)$中抽出样本容量为16的样本,算得其平均数为3160,标准差为100。
试检验假设$H_0:\mu=3140$是否成立($\alpha = 0.01$)。
\end{problem}

\bigskip

\begin{solution}
(1) 待检假设 $H_0 : \mu = 3140$. \dotfill 1分\par
(2) 选取统计量 $T = \frac{\bar{X}-\mu}{S / \sqrt{n}} \sim t(n-1)$. \dotfill 3分 \par
(3) 查表得到 $t_{\alpha} = t_{\alpha} (n - 1) = t_{0.01} (15) =2.947$. \dotfill 5分 \par
(4) 计算统计值 $t = \frac{\bar{x} - \mu_0}{s/\sqrt{n}} =\frac{3160-3140}{100/4} = 0.8$.\dotfill 7分 \par
(5) 由于 $| t | < t_{\alpha}$, 故接受 $H_0$, 即假设成立. \dotfill 8分
\end{solution}

\vfill

\newpagea % A卷分页点

\makepart{证明题}{共~2~小题,每小题~10~分,共~20~分}

\begin{problem}
不使用矩阵可相似对角化的判别定理,直接用矩阵的运算和性质证明下面的矩阵$A
=\left(\begin{array}{cc}
  1 & 1\\
  0 & 1
\end{array}\right)$不能相似对角化,即不存在可逆矩阵$P$和对角阵$\Lambda$使得$P^{-1}AP=\Lambda$。
\end{problem}

\bigskip

\begin{proof}
假设有$P = \left(\begin{array}{cc}
  a & b\\
  c & d
\end{array}\right)$使得$P^{-1}AP = \Lambda$,即$AP=P\Lambda$。\dotfill 2分\par
则有 $$\left(\begin{array}{cc}
  a + c & b + d\\
  c & d
\end{array}\right) = \left(\begin{array}{cc}
  1 & 1\\
  0 & 1
\end{array}\right) \left(\begin{array}{cc}
  a & b\\
  c & d
\end{array}\right) = \left(\begin{array}{cc}
  a & b\\
  c & d
\end{array}\right) \left(\begin{array}{cc}
  \lambda_1 & \\
  & \lambda_2
\end{array}\right) = \left(\begin{array}{cc}
  a \lambda_1 & b \lambda_2\\
  c \lambda_1 & d \lambda_2
\end{array}\right)$$ 因此有 $\left\{ \begin{array}{llll}
  a + c & = & a \lambda_1 & (1)\\
  b + d & = & b \lambda_2 & (2)\\
  c & = & c \lambda_1 & (3)\\
  d & = & d \lambda_2 & (4)
\end{array} \right.$ \dotfill 6分\par
由第1个和第3个方程消去$\lambda_1$,可以得到 $c^2 = 0$ 即 $c=0$;
由第2个和第4个方程消去$\lambda_2$,可以得到 $d^2 = 0$ 即 $d=0$。
因此矩阵$P$不可逆,矛盾。\dotfill 10分
\end{proof}

\vfill

\begin{problem}
设事件$A$和$B$相互独立,证明$A$和$\bar{B}$相互独立。
\end{problem}

\bigskip

\begin{proof}
$P (A \cdot \bar{B}) = P (A - B) = P (A - A B)$ \dotfill 3分 \par
\qquad $= P (A) - P (A B) = P (A) - P (A) P (B)$ \dotfill 6分 \par
\qquad $= P (A) (1 - P (B)) = P (A) P (\bar{B})$ \dotfill 9分 \par
所以$A$和$\bar{B}$相互独立。\dotfill 10分
\end{proof}

\vfill

\makedata{一些可能用到的数据} %附录数据

\begin{tabu}{*{4}{X[l,$]}}
\hline
\Phi_0(0.5)=0.6915 & \Phi_0(1)=0.8413 & \Phi_0(2)=0.9773 & \Phi_0(2.5)=0.9938 \\
t_{0.01}(8)=3.355 & t_{0.01}(9)=3.250 & t_{0.01}(15)=2.947 & t_{0.01}(16)=2.921 \\
\chi_{0.005}^2(8)=22.0 & \chi_{0.005}^2(9)=23.6 & \chi_{0.005}^2(15)=32.8 & \chi_{0.005}^2(16)=34.3 \\
\chi_{0.995}^2(8)=1.34 & \chi_{0.995}^2(9)=1.73 & \chi_{0.995}^2(15)=4.60 & \chi_{0.995}^2(16)=5.14 \\
\hline
\end{tabu}

\end{document}